A mysterious factorial equation.

แชร์
ฝัง
  • เผยแพร่เมื่อ 3 มิ.ย. 2024
  • We solve a nice equation over the natural numbers involving exponentiation and factorials. Our approach employs Wilson's theorem and other modular reduction tricks.
    Please Subscribe: th-cam.com/users/michaelpennma...
    Merch: teespring.com/stores/michael-...
    Personal Website: www.michael-penn.net
    Randolph College Math: www.randolphcollege.edu/mathem...
    Randolph College Math and Science on Facebook: / randolph.science
    Research Gate profile: www.researchgate.net/profile/...
    Google Scholar profile: scholar.google.com/citations?...
    If you are going to use an ad-blocker, considering using brave and tipping me BAT!
    brave.com/sdp793
    Buy textbooks here and help me out: amzn.to/31Bj9ye
    Buy an amazon gift card and help me out: amzn.to/2PComAf
    Books I like:
    Abstract Algebra:
    Judson(online): abstract.ups.edu/
    Judson(print): amzn.to/2Xg92wD
    Dummit and Foote: amzn.to/2zYOrok
    Gallian: amzn.to/2zg4YEo
    Artin: amzn.to/2LQ8l7C
    Differential Forms:
    Bachman: amzn.to/2z9wljH
    Number Theory:
    Crisman(online): math.gordon.edu/ntic/
    Strayer: amzn.to/3bXwLah
    Andrews: amzn.to/2zWlOZ0
    Analysis:
    Abbot: amzn.to/3cwYtuF
    How to think about Analysis: amzn.to/2AIhwVm
    Calculus:
    OpenStax(online): openstax.org/subjects/math
    OpenStax Vol 1: amzn.to/2zlreN8
    OpenStax Vol 2: amzn.to/2TtwoxH
    OpenStax Vol 3: amzn.to/3bPJ3Bn
    My Filming Equipment:
    Camera: amzn.to/3kx2JzE
    Lense: amzn.to/2PFxPXA
    Audio Recorder: amzn.to/2XLzkaZ
    Microphones: amzn.to/3fJED0T
    Lights: amzn.to/2XHxRT0
    White Chalk: amzn.to/3ipu3Oh
    Color Chalk: amzn.to/2XL6eIJ

ความคิดเห็น • 105

  • @prime_suntereh2241
    @prime_suntereh2241 2 ปีที่แล้ว +10

    Wilson + inequality + bionomial + basic congruence = amazing problem

  • @HarmonicEpsilonDelta
    @HarmonicEpsilonDelta 3 ปีที่แล้ว +23

    10:47 I think we used the hypothesis when we multiplied 2*(n/2), because for that to be true, we need both 2 and (n/2) in the product, and for that, we need n/2 to be different than 2, since they come from the factorial, and therefore n>4, which implies n\geq 5.

  • @factsverse9957
    @factsverse9957 3 ปีที่แล้ว +22

    It appeared on the SMO (Singapore Mathematical Olympiad), Open Round 2 on the year 2008, as problem number 1. Hope it helps!

    • @factsverse9957
      @factsverse9957 3 ปีที่แล้ว

      @Adam Romanov You can find the pdf of it online, I got a pdf from a website fadjarp3g, plug in SMO Round 2 2008 fadjarp3g in the Google search bar and I think you should find it, hope this helps

    • @yusrizalakbar3983
      @yusrizalakbar3983 3 ปีที่แล้ว

      14:00 he said "that's true for n≥5" but 4^4 > 4!, why is it
      I'm sorry if it's a dumb questions

    • @yusrizalakbar3983
      @yusrizalakbar3983 3 ปีที่แล้ว

      @qu-ack oh okay thanks

  • @goodplacetostop2973
    @goodplacetostop2973 3 ปีที่แล้ว +73

    12:03
    14:23

    • @caesar_cipher
      @caesar_cipher 3 ปีที่แล้ว +20

      Something very important happened at 12:03 - u must check it out

    • @goodplacetostop2973
      @goodplacetostop2973 3 ปีที่แล้ว +4

      Caesar Cipher Indeed! Thanks 👍

    • @R0M4ur0
      @R0M4ur0 3 ปีที่แล้ว +7

      In general, in this video we have a "that's a good place to stop", quantity squared

    • @goodplacetostart9099
      @goodplacetostart9099 3 ปีที่แล้ว +4

      Good place to start at 0:00

    • @CM63_France
      @CM63_France 3 ปีที่แล้ว +2

      @@R0M4ur0 along with "let's go head and do that","all the way {up|down} to","and so on and so forth"....

  • @roboto12345
    @roboto12345 3 ปีที่แล้ว +21

    I was trying that exact problem, it was in an LTE article. Thanks

  • @ffggddss
    @ffggddss 3 ปีที่แล้ว +25

    First thing I thought when I saw the thumbnail: "Hey, that looks like Wilson's Theorem! . . Oh wait, not quite." Wilson's Theorem says that
    p | [(p-1)! + 1], iff p is prime. Here we would make n = p-1 and this becomes
    (n+1) | (n! + 1), iff (n+1) is prime; vs our problem:
    (n+1)ᵏ = n! + 1
    which is a stronger condition on (n! + 1) than mere divisibility by (n+1).
    k & n are restricted to the +ve integers, so start with k=1:
    N+1 - 1 - n = n!
    Works for n = 1 and 2; no others.
    Solutions: (n,k) = (1,1), (2,1)
    k=2:
    n! + 1 will have to be a square. I think the only cases of that are n = 4, 5, and 7. n = 4 works for this equation; 5 and 7 do not:
    4! + 1 = 25 = 5²
    5! + 1 = 121 = 11²
    7! + 1 = 5041 = 71²
    So:
    (4+1)² - 1 = 4! . . 24 = 24
    Solution: (n,k) = (4,2)
    I suspect there are no others, but I don't see a way to prove that. Let's see what Michael has...
    ... Very nicely done! So my suspicion was borne out. Thanks to Michael for the proof.
    Fred

  • @Kettwiesel25
    @Kettwiesel25 2 ปีที่แล้ว +3

    I wasn't aware of Wilson's Theorem, but it's not too hard to see that n+1 has to be a prime in this problem, this is the easy part of Wilson's Theorem. If n+1 has a factor, which is smaller than it, then it appears in n! as well as (n+1)^k, so the two can not be just 1 apart (they are not coprime)

  • @swayamprakashkar9664
    @swayamprakashkar9664 3 ปีที่แล้ว

    Very nice solution... Nailed it👏👏

  • @mihamihailovic4519
    @mihamihailovic4519 3 ปีที่แล้ว +8

    If it isn't too much of a hastle can you please put the video link in the description for theorems you used and proved in another video (like wilsons theorem here)

  • @a_llama
    @a_llama 3 ปีที่แล้ว +22

    10:47 im guessing n>5 because you assume that there exists an n/2 >= 3, meaning n>=6

    • @UndeadFil
      @UndeadFil 3 ปีที่แล้ว +1

      Correct, and since n+1 has to be prime, the case n=5 can be ruled out because 6 obviously isn't prime.

  • @mariomestre7490
    @mariomestre7490 2 ปีที่แล้ว

    Una equació meravellosa. Genial

  • @mcwulf25
    @mcwulf25 2 ปีที่แล้ว

    Clever solution. Especially noticing n/2 and 2 are factors of n! when n even.

  • @mathissupereasy
    @mathissupereasy 3 ปีที่แล้ว +8

    I would love to see you working on inequality problems.

    • @newkid9807
      @newkid9807 3 ปีที่แล้ว

      Le Family Patriarch
      Bao Quoc Le

  • @angelvalera1997
    @angelvalera1997 3 ปีที่แล้ว

    I love those videos.
    Greetings from Spain :D

  • @prunodagen
    @prunodagen 3 ปีที่แล้ว +14

    You don"t need Wilson Theorem.
    (n+1)^k = n! + 1
    n! + 1 is odd for n > 1, then n + 1 must be odd, then n must be even. (we don't need n + 1 prime, just the fact n is even to get k = 0[n])

    • @Miju001
      @Miju001 3 ปีที่แล้ว

      Woo, that's slick

    • @spiderjerusalem4009
      @spiderjerusalem4009 2 หลายเดือนก่อน

      "just the fact that n is even to get k=0" how?

  • @TwilightBrawl59
    @TwilightBrawl59 3 ปีที่แล้ว +7

    3:25 Not a big mistake but the left hand side is even and the right hand side is odd.

  • @kuldeepparashar7266
    @kuldeepparashar7266 3 ปีที่แล้ว

    Thanks sir

  • @jamesfortune243
    @jamesfortune243 3 ปีที่แล้ว

    Seeing your proofs is making me smarter. :)

  • @pendawarrior
    @pendawarrior 3 ปีที่แล้ว

    Fantastic

  • @math_infinity_
    @math_infinity_ 3 ปีที่แล้ว

    Good question

  • @TheFlash-bw3hi
    @TheFlash-bw3hi 3 ปีที่แล้ว +1

    Woow dope explanation.... Can u please try Putnam 2014 a5 , the official solution is unbearable 😭

  • @thayanithirk1784
    @thayanithirk1784 3 ปีที่แล้ว +2

    Please do Putnam 2006 B2

  • @chhabisarkar9057
    @chhabisarkar9057 3 ปีที่แล้ว +4

    A nice little problem from INMO 2014 , problem 2
    Prove that [n] + [n/2] + [n/3] ....+ [n/n] + [√n] is even , where [n] is the floor of n
    Plz see it . Thank you

    • @Kettwiesel25
      @Kettwiesel25 2 ปีที่แล้ว

      Proof sketch: Notice that f(n) - f(n-1), the increase from n-1 to n, is the number of factors of n, except if n is a square number, then it is 1 more. Now use that a number has an odd number of factors if and only if it is a square.

    • @TechToppers
      @TechToppers 2 ปีที่แล้ว

      Hello 😂

    • @chhabisarkar9057
      @chhabisarkar9057 2 ปีที่แล้ว

      @@TechToppers hmmmm

    • @spiderjerusalem4009
      @spiderjerusalem4009 2 หลายเดือนก่อน

      Let f(n) be the above expression
      f(n-1)
      =Σⁿ⁻¹ₖ₌₁ ⌊ⁿ⁻¹/ₖ⌋ + ⌊√(n-1)⌋
      = Σⁿ⁻¹ₖ₌₁ (⌈ⁿ/ₖ⌉-1) + ⌊√(n-1)⌋
      (Here, we used the fact that ⌈ⁿ/ₖ⌉=⌊ⁿ⁻¹/ₖ⌋+1 for any n,k∈ℕ)
      f(n)-f(n-1)
      = n-Σⁿₖ₌₁ (⌈ⁿ/ₖ⌉-⌊ⁿ/ₖ⌋)
      +⌊√n⌋-⌊√(n-1)⌋
      Let Δ(k,n)=1 if k|n, 0 otherwise
      hence ⌈ⁿ/ₖ⌉-⌊ⁿ/ₖ⌋=1-Δ(k,n)
      summing from k=1 to n, it equals
      Σⁿₖ₌₁ (1-Δ(k,n))
      = n-Σₖₗₙ 1 = n-τ(n)
      Thus
      f(n)-f(n-1)=τ(n)+⌊√n⌋-⌊√(n-1)⌋
      •》Case 1: n is perfect square
      implying ⌊√n⌋=1+⌊√(n-1)⌋
      hence f(n)-f(n-1)=τ(n)+1, which is even since τ(n) is odd if and only if n is a perfect square
      •》n is not perfect square
      implying ⌊√n⌋=⌊√(n-1)⌋
      hence f(n)-f(n-1)=τ(n), which ia even
      in either case, f(n)-f(n-1)=k
      for some even k
      summing from 2 to n
      f(n)=(n-1)k+f(1)
      = (n-1)k+2
      = 2((n-1)(k/2)+1)
      which is even for any n∈ℕ, as desired.

  • @sylviabayard6117
    @sylviabayard6117 3 ปีที่แล้ว +6

    A way to skip Wilson's Theorem:
    Add 1 to both sides, to get
    (n+1)^k = n! + 1
    Well, we know that n! + 1 is coprime to any number less than or equal to n, so any primes dividing (n+1)^k must be greater than or equal to n+1. However, by the very form of it, any primes dividing (n+1)^k must be less than or equal to n+1.
    Therefore the ONLY prime dividing (n+1)^k must be n+1.

    • @factsverse9957
      @factsverse9957 3 ปีที่แล้ว

      Then? How can we proceed, it's unclear for me

    • @blackmephistopheles2273
      @blackmephistopheles2273 3 ปีที่แล้ว

      @@factsverse9957 Continue the argument in the video, after the point where Wilson's Theorem was used; this argument above shows that WT is unnecessary.
      So, continue from 6:56 (edited to find time reference).

    • @Pedro-fg4sw
      @Pedro-fg4sw 3 ปีที่แล้ว

      LTE Lemma

    • @spiderjerusalem4009
      @spiderjerusalem4009 2 หลายเดือนก่อน

      but... that is just a rephrase of the converse of wilson's theorem which was applied in the vid....

  • @abhishekbhattacharjee4039
    @abhishekbhattacharjee4039 3 ปีที่แล้ว

    This problem is beautiful

  • @hans-juergenbrasch3683
    @hans-juergenbrasch3683 3 ปีที่แล้ว

    n=1,2 imply k=1. Now for n>2 we must have n+1=p is a prime, otherwise n+1 would have a proper divisor 1 < d =< (n+1)/2 < n which yields a contradiction modulo d:
    0=n!=(n+1)^k-1=-1 and hence we can rewrite p^k-1=(p-1)! with p>=5 or
    1+p+...+p^(k-1)=(p-2)! Note that gcd(1+p+...+p^(k-1),p-1)=gcd(k,p-1)
    Since (p-1)/2 < p-2 we must have
    k = t (p-1)/2 for some t. Since p^(p-1)-1=
    = (1+(p-1))^(p-1)-1>(p-1)^(p-1)
    we must have t=1, i.e.
    p^((p-1)/2)-1=(p-1)!
    For p=5 we get a solution corresponding to n=4, k=2 and for p>5 we get p^((p-1)/2)-1=
    =prod_(i=1)^((p-1)/2) [i(p-i)]>
    >[p^2+(p-3)^2-5]p^[(p-1)/2)-2]>
    >p^((p-1)/2), a contradiction. Note, that we have split the product into two factors, one for i=1,2 given by (p-1)2 (p-2) and the other for i=3,..., (p-1)/2

  • @joaopedrobmenezes2977
    @joaopedrobmenezes2977 3 ปีที่แล้ว

    Vp(x!) >= 2*Vp(x) for all non prime x such that x> 4, then you can see that if n>4 and it is not prime( which is the case for n >4) k would have to be at least n, but (n + 1)^n - 1 > n! for n > 4. This is just an alternative solution:)

  • @TechToppers
    @TechToppers 3 ปีที่แล้ว

    That was brain storming!
    Uff
    Will see again...

  • @WriteRightMathNation
    @WriteRightMathNation 3 ปีที่แล้ว

    Nice problem, but at 3:28 (someone else may have already pointed this out...), you mixed up "left-hand" and "right-hand"... 😊

  • @TechToppers
    @TechToppers 3 ปีที่แล้ว +1

    Let s(n) denote the sum of the digits of a positive integer n in base 10. If s(m) = 20 and s(33m) = 120,
    what is the value of s(3m)?
    Can someone please help me with this?
    It came in India. PRMO 2019 25th August Paper

  • @stevenwilson5556
    @stevenwilson5556 3 ปีที่แล้ว

    6:53 why is n + 1 a prime? Does this cover cases where n + 1 is not prime?

  • @gregsavitt7176
    @gregsavitt7176 3 ปีที่แล้ว +2

    Just got out of Calc 3 thinking that I had gotten pretty good at math. Then I watched this and my brain melted.

    • @MysticMagik
      @MysticMagik 3 ปีที่แล้ว +1

      This is mere child's play compared with calc 3

    • @spiderjerusalem4009
      @spiderjerusalem4009 2 หลายเดือนก่อน

      ​@@MysticMagikquite the opposite. Lots of calc stuffs are straight-up chug-in-plug-in dumb play, whilst number theory and combinatorics are whole different unpredictable beast

  • @user-A168
    @user-A168 3 ปีที่แล้ว

    Good

  • @donaldbiden7927
    @donaldbiden7927 3 ปีที่แล้ว

    Beautiful solution ! Kindly tell the motivation for the inequality.

    • @ericslavich4297
      @ericslavich4297 3 ปีที่แล้ว

      Once we know k is a multiple of n, you look at the original equation again. You realize the exponent on the left hand side is at least n. Also note n^n > n!, so the left hand side is growing more rapidly than the right. The rest is details and being rigorous.

  • @masonholcombe3327
    @masonholcombe3327 3 ปีที่แล้ว +2

    Video idea: Prove the Hardy-Ramanajan Series for partitions of n :)

    • @geometrydashmega238
      @geometrydashmega238 3 ปีที่แล้ว

      He has a playlist of partitions and I think he was going to do that, or did it already. Check that out

  • @subversively6680
    @subversively6680 3 ปีที่แล้ว

    By applying Wilson 's theorem, can we find the rest of prime numbers that are >100 (theoretically speaking)

    • @ruairigarrett6834
      @ruairigarrett6834 3 ปีที่แล้ว +1

      Good luck calculating those factorials lol

    • @iabervon
      @iabervon 3 ปีที่แล้ว

      @@ruairigarrett6834 You only need them mod p+1, so at least you're not dealing with numbers more than about p², but you're still doing p multiplications and might as well just try dividing p by everything less than p.

  • @Polpaccio
    @Polpaccio ปีที่แล้ว

    1 year later , 200k more subs

  • @LiangLao2
    @LiangLao2 3 ปีที่แล้ว +1

    just a suggestion, can you open a new channel? one channel focuses on math question especially the Olympic question, one channel focus math theory such as analysis algebra number theory, etc.

  • @accountname1047
    @accountname1047 3 ปีที่แล้ว

    The form of the numbers is similar to the brown numbers

  • @rustemtehmezov9494
    @rustemtehmezov9494 3 ปีที่แล้ว

    Where is guy says everytime "answer=1"?

  • @jideofororamah4399
    @jideofororamah4399 3 ปีที่แล้ว +1

    Because K is the power and n=2 (n+1)^k = n!. 1 is a constant so how did you get k=1 where n=2

    • @RealLifeKyurem
      @RealLifeKyurem 3 ปีที่แล้ว +1

      3^k = 3
      What k satisfies this equation?

    • @minh9545
      @minh9545 3 ปีที่แล้ว

      @@RealLifeKyurem what do the 3 stripes and the -1(mod p) mean?

    • @RealLifeKyurem
      @RealLifeKyurem 3 ปีที่แล้ว +1

      TNT ≡ is read “is congruent to.” -1 mod p means if you divide a number n by a prime p, the remainder is -1. In other words, it satisfies the expression:
      apm - 1; for any integer a.
      The reason why ≡ is used instead of =, is because any multiple of p satisfies the expression.

  • @xinfuchen5355
    @xinfuchen5355 3 ปีที่แล้ว +1

    No Wilsons Theorem is needed. When n>4,
    i) (n+1)^k= 1+n!= 1 mod (2). This imples that n is even
    ii) n even implies that (n-1)! = 0 mod (n)
    iii) k = ((n+1)^k-1)/n mod(n) = (n-1)! mod (n) = 0 mod(n)
    (iva) k=0, no
    (ivb) k>=n, Then (n+1)^k > n^n -1> n!-1. No solution,
    Hence, when n>4, there is no solution.

    • @Teodyk
      @Teodyk 2 ปีที่แล้ว

      Obviously.

  • @tomholroyd7519
    @tomholroyd7519 3 ปีที่แล้ว

    Count Olaf

  • @seroujghazarian6343
    @seroujghazarian6343 3 ปีที่แล้ว +1

    For integers, n^n=>n! Is true. But beyond that, it's not always true. Case in point:(1/2)^(1/2)=sqrt(2)/2 < (1/2)! = sqrt(pi)/2

  • @devrajdas6544
    @devrajdas6544 3 ปีที่แล้ว +2

    Is there any problem you can't solve?

  • @user-vk6nl6mi1d
    @user-vk6nl6mi1d 2 ปีที่แล้ว +1

    0 not a natural number?

  • @mcbeaulieu
    @mcbeaulieu 3 ปีที่แล้ว

    What is Mod? 😭😭😭😭

    • @kruksog
      @kruksog 3 ปีที่แล้ว

      Mod gives remainder. Some examples: 5 mod 3 = 2 because 5/3 has remainder 2. 4 mod 2 = 0 because 4 is divisible by 2 (i.e. no remainder). Once again, 7 mod 2 = 1 because 7/2 has a remainder of 1.
      It's important to remember when doing elementary number theory, we work in the integers and naturals. Hence all the whole number concerns.

  • @rustemtehmezov9494
    @rustemtehmezov9494 3 ปีที่แล้ว

    You get (n+1)^mn-1>nⁿ>n! but isn't it was (n+1)^mn-1=n! and this means n!>nⁿ? İ think it was typo

    • @oussemabouaneni992
      @oussemabouaneni992 3 ปีที่แล้ว

      That's the contradiction. He supposed that such an n exists and proved that its existence is impossible.

    • @rustemtehmezov9494
      @rustemtehmezov9494 3 ปีที่แล้ว

      @@oussemabouaneni992 but if we get n!>nⁿ then this will be contradiction, because this never holds for these rules. But he got nⁿ>n! which is clearly true. He should be write n!>nⁿ (he got it at the end but wrote it wrongly i think, but i understood his proof method clear)

    • @oussemabouaneni992
      @oussemabouaneni992 3 ปีที่แล้ว +1

      @@rustemtehmezov9494 No no he wrote it perfectly correctly. He proved that (n+1)^k - 1 > n^n > n! which means that (n+1)^k - 1 is never equal to n! for n >= 5 which is what he set out to prove.

    • @rustemtehmezov9494
      @rustemtehmezov9494 3 ปีที่แล้ว +1

      @@oussemabouaneni992 Oh, now i understood what you mean, yeah this is accetable. (Almost same as i supposed) Thanks for clarification👍

    • @oussemabouaneni992
      @oussemabouaneni992 3 ปีที่แล้ว

      @@rustemtehmezov9494 Cheers mate.

  • @xinfuchen5355
    @xinfuchen5355 3 ปีที่แล้ว

    No Wilson's Theorem is needed. When n>4,
    i) (n+1)^k= 1+n!= 1 mod (2). This implies that n is even
    ii) n even and n>4 imply that (n-1)! = 0 mod (n)
    iii) k = ((n+1)^k-1)/n mod(n) = (n-1)! mod (n) = 0 mod(n)
    (iva) k=0, no
    (ivb) k>=n, Then (n+1)^k-1 >=( n+1)^n -1>=n^n>n!. No solution,
    Hence, when n>4, there is no solution.

  • @mandelbrotsugee
    @mandelbrotsugee 3 ปีที่แล้ว

    じっと見ていると、n=4と分かりますな。
    K=2

  • @tyjensen8366
    @tyjensen8366 3 ปีที่แล้ว +3

    God he’s such a chad

    • @guibaroleo
      @guibaroleo 3 ปีที่แล้ว

      hahahahahaha

  • @HagenvonEitzen
    @HagenvonEitzen 3 ปีที่แล้ว

    13:33 Or just (n+1)^n -1 >= (n+1)n(n-1)... 2 - =(n+1)! -1 > n!

  • @sdspivey
    @sdspivey 3 ปีที่แล้ว

    You say K must be a multiple of N, but the solutions given have N as a multiple of K.

    • @axelperezmachado3500
      @axelperezmachado3500 3 ปีที่แล้ว

      all solutions with n >= 5 must have k as a multiple of n. That does not necessarily apply to the given solutions.

  • @jideofororamah4399
    @jideofororamah4399 3 ปีที่แล้ว +1

    I got confused at how you got k=1 at n=2

    • @ddp8511
      @ddp8511 3 ปีที่แล้ว +3

      Just put n=2
      U get 3^k=3

    • @jideofororamah4399
      @jideofororamah4399 3 ปีที่แล้ว +1

      Angel Mendez-Rivera That solves it. Thanks 🙏❤️ @michael penn

  • @keithmasumoto9698
    @keithmasumoto9698 3 ปีที่แล้ว

    Immediately I don't get why n+1 is prime. If n=5 then n+1=6 is not prime.

    • @RandomGamer9
      @RandomGamer9 3 ปีที่แล้ว

      I had to rewatch to understand it but if you reduce (n+1)^k-1=n! (mod n+1) you get 0-1=n! (mod n+1) then you rearrange to get the form from Wilson's theorem and that n+1 is prime. The zero in the above equation come from the fact that (n+1)^k is divisible by (n+1)

  • @CM63_France
    @CM63_France 3 ปีที่แล้ว +2

    Hi,
    This was before hair cutting, so I understand my request about the camera is not taken into account.